LSAT and Law School Admissions Forum

Get expert LSAT preparation and law school admissions advice from PowerScore Test Preparation.

 Administrator
PowerScore Staff
  • PowerScore Staff
  • Posts: 8929
  • Joined: Feb 02, 2011
|
#25860
Complete Question Explanation

Assumption—SN. The correct answer choice is (D)

After a slight break in Question 15, a mid-section trend of long stimuli, which began in Question 14, will continue through Question 18. As with Question 14, however, the length of this stimulus masks an otherwise straightforward argument. And, while there are several conditional relationships in the stimulus, you can readily identify the logical flaw without resorting to diagramming those relationships.

Here, the professor expresses concern about funding for the chemistry department’s research. Currently, pharmaceutical companies and other profit-driven institutions provide nearly all of the funding. The professor states that, unless the chemistry department can secure more funding for basic science research, it is highly unlikely that any significant advances in basic research will come out of the department. Based on this fact, the professor concludes that without increased funding from sources other than profit-driven institutions, the chemistry department is unlikely to gain the prestige that only achievements in basic science research confer.

The stimulus is very wordy, but the argument is simple. The chemistry department needs more funding to perform the basic science research that could gain the department prestige. The current funding comes from pharmaceutical companies and profit-driven institutions. So, to do the research needed to gain prestige, the department needs to increase funding from sources other than profit-driven institutions.

But why is it necessary for the funding to come from institutions that are not profit-driven? The argument offers no support for this shift in funding source being required. For example, we are not told that there is no more for-profit funding available. So, this transition in the conclusion without support is a logical flaw.

The question stem identifies this as an Assumption question. Your prephrase is that the correct answer choice will likely defend the conclusion against the possibility that there is more funding available from the profit-driven sources.

Answer choice (A): Remember that the correct answer choice is necessary for the conclusion to be valid. This choice presents a reversal of the evidence presented in the stimulus, and runs counter to the conclusion. More funding was necessary to conduct the research that could lead to significant advances, while this answer choice treats additional funding as sufficient to produce significant advances.

Answer choice (B): As with choice (A), this choice depends on a factual situation that runs counter to the conclusion, in which the professor stated the chemistry department is unlikely to gain prestige. Also, this scenario reverses the relationship described in the scenario, in which funding was needed to increase prestige.

Answer choice (C): Again, the information in this choice is inconsistent with the professor’s argument, in which he discussed the need for additional funding, as opposed to stating the need to remove current funding sources. The problem the professor associated with the current funding was not that too much of it came from profit-driven sources, but rather that more funding was needed than could be obtained from the profit-driven sources.

Answer choice (D): This is the correct answer choice. This choice makes express the idea implicit in the professor’s argument that funding from the profit-driven sources is unlikely to increase. If this were not the case, and the chemistry department’s funding were likely to increase even if the funding from sources other than profit-driven institutions did not increase, then the conclusion in the stimulus would be invalid.

Answer choice (E): Since nothing in the stimulus implied that the existence of some benefit to the funding sources was necessary for the funding levels to increase, this choice is incorrect.
 Kp13
  • Posts: 32
  • Joined: Jun 17, 2013
|
#13171
Hi,

I am having a hard time differentiating between the chosen wrong answer A and the correct answer D.

Thank you,
 Nikki Siclunov
PowerScore Staff
  • PowerScore Staff
  • Posts: 1362
  • Joined: Aug 02, 2011
|
#13195
Hi Kp13,

Answer choice (A) is a Mistaken Reversal of a premise. When simplified, the argument is structured as follows:

Premise: Advances in basic research :arrow: More money

Premise: Gain prestige :arrow: Advances in basic research

Conclusion: Gain prestige :arrow: Money from sources other than Big Pharma.

There is a gap in the pattern of reasoning here. Just because advances in basic research require more funding doesn't mean that the additional funding should come from sources other than Big Pharma. Why not get the money from the same sources as before (i.e. Big Pharma)? The author is assuming that this is impossible, i.e. that additional funding can come only from sources other than Big Pharma. This is essentially the contrapositive of answer choice (D):

Money from sources other than Big Pharma :arrow: More money

CP: More money :arrow: Money from sources other than Big Pharma

By comparison, answer choice (A) states:

More money :arrow: Advances in basic research

This is clearly a Mistaken Reversal of the first premise, making answer choice (A) incorrect.

Hope this helps! Let me know.
 Kp13
  • Posts: 32
  • Joined: Jun 17, 2013
|
#13218
Thanks for breaking it down for me Nikki. It's a lot clearer now.
 mankariousc
  • Posts: 32
  • Joined: Feb 13, 2017
|
#32749
While I see D as the better answer, I am still struggling to understand why A is wrong. Does it have to do with the idea that the funding should be from the pharmaceutical companies/other profit-driven institutions specifically?

Thanks!
 Adam Tyson
PowerScore Staff
  • PowerScore Staff
  • Posts: 5190
  • Joined: Apr 14, 2011
|
#32763
Weclome to the forum, mankariousc, and thanks for asking! As Nikki explained previously in this thread, answer A is a Mistaken Reversal of one of the premises. It's a little tricky to see at first, perhaps, because that premise used the necessary indicator "unless", which may require some mental gymnastics to diagram conditionally.

Unless indicates the necessary condition - in this case, that condition is getting more funding. The other thing in that relationship (advances are unlikely) gets negated, and that negated term (advance are likely) becomes your sufficient condition. So, that premise diagrams as Nikki showed - if advances are likely, we got more funding. Don't worry about the likely vs. certain aspect of it for now - the two are close enough in this case to illustrate the problem.

We can prove the contrapositive of a conditional claim. That's where we reverse the order of the terms and also negate them both. So, we can be sure that if we do not get more funding, we won't likely get advances. Two common errors are the Mistaken Negation (where you simply negate the terms but leave the order alone: if we aren't likely to make advances then we must not be getting more funding) and the Mistake Reversal (where you change the order but don't negate: we got more funding so we are likely to make advances), and while those are possible, they cannot be proven.

Imagine this for comparison: If you get into Harvard it's likely you got a 170+ on the LSAT. What can I prove? If you did not get a 170+ on the LSAT then you probably didn't get into Harvard.

The Mistaken Negation would be: if you did not get into Harvard then you probably did not get a 170+. Possible, but not provable - maybe you got a 180, but you simply didn't apply to Harvard? Maybe you had too low a GPA? Maybe you got caught lying on your application, or your Letters of Recommendation were awful?

The Mistake Reversal would be: you got a 170+ so you probably got into Harvard. Again, possible, but not provable, for all the same reasons as above.

Answer A is making that Mistaken Reversal, and that's why it is not something that the author must have assumed here. He assumed the contrapositive of the original claim, but he need not assume the mistaken negation or reversal.

I hope that helps! Conditional reasoning is tricky at first, but with some study and practice it will become much more natural and obvious, and both your LR and LG performance will likely improve as a result. Good luck, and keep coming back here for more!
 mankariousc
  • Posts: 32
  • Joined: Feb 13, 2017
|
#32767
This makes a lot more sense, Adam! Thank you so much! I understand the mistaken negation/reversal a lot better now. Could you explain though how you would diagram the correct answer (D)?
 Adam Tyson
PowerScore Staff
  • PowerScore Staff
  • Posts: 5190
  • Joined: Apr 14, 2011
|
#32768
I'd be happy to do so, but I don't think I could do it any better than Nikki already did. Scroll back up to his explanation and you'll see his conditional diagrams, and the gap in the argument between the premises about "more money" and the conclusion about "other sources". Basically, you need to supply that missing link, which is essentially:

More Money :arrow: Other Sources

Answer D provides the contrapositive of that link, which is just as good:

Other Sources :arrow: More Money

Glad to help!
 Khodi7531
  • Posts: 116
  • Joined: Mar 14, 2018
|
#44915
So I can mostly see why D is correct... A was wrong because it was an illegal negation which I noticed. However, I chose E. I think it's because I anticipated it to be too similar to this answer and went with it.


In E, I circled NOT LIKELY TO BENEFIT because I really didn't like it. For a nec assumption I was thinking this is not something that's necessary. But i'm just getting back into the LSAT so i'm ok with making this mistake right now - cause I know it won't persist.


But my anticipated response was something like, "profit institutions (pharm) do not fund basic research (or enough at least). "not funding" may be too strong but I was thinking something like that.

Now I saw what was wrong with it generally...but this thought led me to E and half crossed D.

Also on a side note....for nec assumptions not sure if this is a good methodology but when I read it I see some gaps just by the words. Like the jump and knowing that it'll have to do with one word and another (in this case being the basic research and pharm/for profit companies). Once I think a little more I can see what comes "first and second" so I don't negate incorrectly but is this signs of a intuitive or gut understanding? Because the more I try to break it down and not go with this method the longer it takes
 Emily Haney-Caron
PowerScore Staff
  • PowerScore Staff
  • Posts: 577
  • Joined: Jan 12, 2012
|
#45030
Hi Khodi,

It's great to have a gut instinct with these kinds of questions about what your pre-phrase might be! It is critical, though, to check that gut instinct against the techniques you've learned to make sure it matches all the information you have available. I generally recommend against going on gut instinct alone, because even when it is mostly right, usually it is wrong at least some of the time. :) It is awesome that these questions come easier to you, though, and that gives you something to build on as you're learning specific strategies.

Get the most out of your LSAT Prep Plus subscription.

Analyze and track your performance with our Testing and Analytics Package.